RegistrierenRegistrieren   LoginLogin   FAQFAQ    SuchenSuchen   
Teilchen im harmonischen Potential
 
Neue Frage »
Antworten »
    Foren-Übersicht -> Wärmelehre
Autor Nachricht
wanoek



Anmeldungsdatum: 17.04.2007
Beiträge: 18

Beitrag wanoek Verfasst am: 17. Apr 2007 21:22    Titel: Teilchen im harmonischen Potential Antworten mit Zitat

hallo physiker ... ich habe eine aufgabe, für die mir irgendwie der ansatz fehlt

also man soll eine Verteilungsfunktion berechnen für das system mit dem dreidimensionalen oszillatorpotential

es gibt nun diese formel ...

erstens ist es der richtige anstatz ? ... wenn ja setze ich dann das potential für im exponenten ein ? und was mache ich dann ?
sax



Anmeldungsdatum: 10.05.2005
Beiträge: 377
Wohnort: Magdeburg

Beitrag sax Verfasst am: 18. Apr 2007 00:31    Titel: Antworten mit Zitat

Tja, das kommt darauf an, was für eine Gesamtheit du betrachtest,
im kanonischem Ensemble ist dein Ansatz richtig. Im mikrokanonischen oder im großkanonischen nicht.
Wie lautet denn die vollständige Aufgabe ?
wanoek



Anmeldungsdatum: 17.04.2007
Beiträge: 18

Beitrag wanoek Verfasst am: 18. Apr 2007 11:48    Titel: Antworten mit Zitat

na ja ... was für ein ensemble das jetzt ist ... da hab ich keine ahnung, jedenfalls lautet die aufgabe wörtlich so :

teilchen im harmonischen potential

gegeben sei ein system aus nicht-wechselwirkenden Teilchen der Masse , die sich in einem dreidimensionalen oszillatorpotential befinden.

a) wie lautet die verteilungsfunktion für dieses system.
hinweis: die verteilungsfunktion muss die normierungsbedingung erfüllen.
wanoek



Anmeldungsdatum: 17.04.2007
Beiträge: 18

Beitrag wanoek Verfasst am: 18. Apr 2007 13:31    Titel: Antworten mit Zitat

also .. mein ansatz war für die lösung der aufgabe wohl richtig .. hätte es dann einfach ausrechnen müssen ...
also :

mit der formel

setzt man das potential ein bekommt man :

so nun soll ja die funtion die normierungsbedingung erfüllen, also:

und mit der formel, ergibt sich dann

jetzt "einfach" integrieren Augenzwinkern und man hat schließlich sowas

(es gibt wohl einen wert für dieses integral für -unendlich bis unendlich, der dann so aussieht)
jetzt weiter zusammenfassen und man kriegt letzendlich


na ja .. ich wäre dann wohl bei dem integral von e hoch x quadrat gescheitert ...
sax



Anmeldungsdatum: 10.05.2005
Beiträge: 377
Wohnort: Magdeburg

Beitrag sax Verfasst am: 19. Apr 2007 02:05    Titel: Antworten mit Zitat

Sorry ich hatte heute keine Zeit hier reinzuschauen. Aber du hast es ja selbst gelöst, bzw. die Lösung bekommen.
Das integral

und seine Variationen ist übrigens eins der wichtigsten Integrale,
es wird dir, insbesondere in der statistischen Physik, ständig über den Weg laufen, eine der Formel die man wissen sollte, wenn man nachts um drei geweckt wird. Big Laugh
wanoek



Anmeldungsdatum: 17.04.2007
Beiträge: 18

Beitrag wanoek Verfasst am: 19. Apr 2007 16:26    Titel: Antworten mit Zitat

jo .. jetzt kenn ich die Augenzwinkern
Neue Frage »
Antworten »
    Foren-Übersicht -> Wärmelehre